4 svar
70 visningar
hejmo behöver inte mer hjälp
hejmo 85
Postad: 5 sep 19:34

konvergent eller divergent ?

Hej jag behöver lite hjälp med att första vad konvergent och divergent innebär. Såsom jag har förstått det så konvergerar en serie om det existerar ett gränsvärde när k→∞. 

Jag har följande serie som jag ska ta reda på om den konvergerar eller konvergerar absolut.

Jag skriver om och får att k=1ln(k)ln(e)+2ln(k)=ln(k)2ln(k)=12

Facit säger att serien divergerar. Hur kommer det sig att den divergerar trots att den närmar sig 1/2 när k→∞?

Smutsmunnen 1048
Postad: 5 sep 19:37
hejmo skrev:

Hej jag behöver lite hjälp med att första vad konvergent och divergent innebär. Såsom jag har förstått det så konvergerar en serie om det existerar ett gränsvärde när k→∞. 

Jag har följande serie som jag ska ta reda på om den konvergerar eller konvergerar absolut.

Jag skriver om och får att k=1ln(k)ln(e)+2ln(k)=ln(k)2ln(k)=12

Facit säger att serien divergerar. Hur kommer det sig att den divergerar trots att den närmar sig 1/2 när k→∞?

Vart försvinner summatecknet? Sen behöver du tänka igenom hur logaritmreglerna funkar.

hejmo 85
Postad: 5 sep 19:46

förlåt glömde skriva in lim, det ska stå limkln(k)2ln(k)=12

Laguna Online 30059
Postad: 5 sep 19:59

Det där uttrycket är lika med 1/2 även utan limes, men brydde du dig om det som Smutsmunnen skrev?

Moffen 1875
Postad: 5 sep 22:42 Redigerad: 5 sep 22:44

Hej,

Det gäller att skilja på vad som händer med termerna då kk ökar och vad som händer med summan. Självklart har du rätt i att gränsvärdet av termen är lika med en halv, men vad innebär det då för summan om vi summerar ihop en massa halvor? Eller till och med fler än bara en massor?

Jag skulle vilja formulera divergensen på följande vis:

Ändliga summor är inga problem för konvergens, så det enda intressanta är vad som händer för stora kk. Betrakta därför istället summan S1=k=N+lnklne+k2\displaystyle S_1=\sum_{k=N}^{+\infty}\frac{\ln\left(k\right)}{\ln\left(e+k^2\right)} för något k>Nk>N sådant att (motivera varför ett sådant finns):

lne+k2lnk2+k2=ln2k2lnk3=3lnk\displaystyle\ln\left(e+k^2\right)\leq \ln\left(k^2+k^2\right)=\ln\left(2k^2\right)\leq\ln\left(k^3\right)=3\ln\left(k\right).

Då gäller att din summa S1S_1 är större än summan S2=k=N+lnk3lnk=k=N+13+\displaystyle S_2=\sum_{k=N}^{+\infty}\dfrac{\ln\left(k\right)}{3\ln\left(k\right)}=\sum_{k=N}^{+\infty}\dfrac{1}{3}\to+\infty, som tydligt divergerar.

Slutligen är givetvis din summa i uppgiften större än S1S_1 då vi bara har struntat i dom första NN termerna, vilka alla är större än noll.

Svara
Close